LSAT and Law School Admissions Forum

Get expert LSAT preparation and law school admissions advice from PowerScore Test Preparation.

 Administrator
PowerScore Staff
  • PowerScore Staff
  • Posts: 8949
  • Joined: Feb 02, 2011
|
#26530
Complete Question Explanation

Weaken—CE. The correct answer choice is (B)

The conclusion in this stimulus is that there is another, unknown planet that is pulling Uranus away from the Sun and inner planets. In classic causal-weaken style, the correct answer choice presents an alternate cause to explain why Uranus is being tugged in such a way.

Answer choice (A): The year in which Pluto was discovered has nothing to do with the cause of the force being exerted on Uranus.

Answer choice (B): This is the correct answer choice. The belt of comets described could be gravitational force that is pulling on Uranus, and therefore it would not be another planet as the conclusion suggests.

Answer choice (C): The fact that both Neptune and Pluto are smaller than Uranus does not attack the idea that there is an undiscovered planet pulling on Uranus. However, if you don't think the test makers have a sense of humor, read this answer in an almost literal sense :-D

Answer choice (D): This answer choice is tempting, but the stimulus says that a force is tugging on Uranus, not that Uranus is drifting away from the Sun. So you must account for the pull of something else, which this answer choice does not do.

Answer choice (E): The proximity of Neptune’s and Pluto’s orbits do not attack the fact that the author believes another planet is pulling on Uranus.
 ginapark96
  • Posts: 5
  • Joined: May 24, 2017
|
#36148
I eliminated answer choice B because this states the comets are beyond the orbit of Pluto, whereas the stimulus states Uranus is the one being pulled away from the Sun. Hence, I chose D because it was an alternate cause but still referring to Uranus, not Pluto. Could you please help with my flaw in reasoning? Thank you!
User avatar
 Dave Killoran
PowerScore Staff
  • PowerScore Staff
  • Posts: 5978
  • Joined: Mar 25, 2011
|
#36149
Hi Gina,

Thanks for the question! This has always been one of my favorites since it shows how specific they can be with the "tugged" away" idea vs just drifting away. but more on that in a second.

In the case of answer choice (B), the idea here is that although the belt of comets is beyond Pluto, since it has a "powerful gravitational pull" then it could be strong enough to be what is pulling Uranus away from the sun. So, with that being the case, it could be the belt of comets causing Uranus to pull away, not some undiscovered planet. that's a classic alternate cause attack, and thus (B) weakens the argument.

The problem with (D) comes down to the stimulus language of a force pulling Uranus away. This answer indicates that the Sun is allowing Uranus to drift away from it, but that's a different idea than being pulled away. Faced with this answer, the author would simply say, "That may be, but Uranus is being pulled away by some force, and I still think it's an undiscovered planet that is the cause of that." In other words, (D) doesn't make the author change his or her position, and so d=(D) does not weaken the argument.

Please let me know if that helps. Thanks!
 ginapark96
  • Posts: 5
  • Joined: May 24, 2017
|
#36175
That makes sense now. Thank you so much for your help!

Get the most out of your LSAT Prep Plus subscription.

Analyze and track your performance with our Testing and Analytics Package.